4
$\begingroup$

Given any $n\in\mathbb N$, consider the the Sylvester-Hadamard-Walsh matrix $M=(a_{i,j})_{i,j\in 2^n}$ of size $2^n\times 2^n$ and for a number $p\in[1,\infty)$, let $$\nu_{n,p}=\max_{F\subseteq 2^n}\Big(\sum_{j\in 2^n}\big|\sum_{i\in F}a_{i,j}\big|^p\Big)^{1/p}\quad\mbox{and}\quad \tilde \nu_{n,p}=\frac1{2^{2^n}}\sum_{F\subseteq 2^n}\Big(\sum_{j\in 2^n}\big|\sum_{i\in F}a_{i,j}\big|^p\Big)^{1/p}.$$ For $p=2$, the Pithagoras Theorem and the orthogonality of the rows of the matrix $M$ imply that $\nu_{n,2}=2^n$. Using this equality, it is easy to show that $\nu_{n,p}=2^n$ for all $p\in[2,\infty)$.

If $p\in[1,2]$, then by the Holder inequality, we obtain $$2^n\le\nu_{n,p}\le 2^{n(\frac1p+\frac12)}.$$ In particular, $2^n\le\nu_{n,1}\le 2^{3n/2}$. On the other hand, computer calculations show that $\tilde \nu_{n,1}$ and $\nu_{n,1}$ are much smaller than $2^{3n/2}$ (the values of $\tilde\nu_{n,1}$ are calculated using the formula $$\tilde\nu_{n,1}=\frac1{2^{2^n}}\Big(\sum_{i=0}^{2^n}{2^n\choose i}i+2(2^n-1)\sum_{0\le i<j\le 2^n}{2^n\choose i}{2^n\choose j}(j-i)\Big)$$ of Alex Ravsky suggested in his comment): $$ \begin{array}{c|c|c|c|c|c|c} n&2^n&\tilde \nu_{n,1}&\nu_{n,1}&\lfloor 2^{3n/2}\rfloor&\tilde\nu_{n,1}/2^{3n/2}\\ \hline 0&1&1&1&1&1\\ 1&2&1.5&2&2&0.53...\\ 2&4&4.25&6&8&0.53...\\ 3&8&11.65...&14&22&0.51...\\ 4&16&31.56...&40&64&0.49...\\ 5&32&85.41...&\ge 96&181&0.47...\\ 6&64&232.28..&??&512&0.45...\\ 7&128&636.09...&??&1448&0.43...\\ 8&256&1754.09...&??&4096&0.42...\\ 9&512&4866.56...&??&11585&0.42...\\ \end{array} $$

Problem 1. Is $\nu_{n,1}\ge\frac12 2^{3n/2}$? Is $\tilde\nu_{n,1}\ge \varepsilon 2^{3n/2}$ for some $\varepsilon>0$?

Problem 2. Is $\tilde\nu_{n,1}=o(2^{3n/2})$? Is $\nu_{n,1}=o(2^{3n/2})$?

Problem 3. Find nontrivial lower and upper bounds on the number $$\lambda_1=\limsup_{n\to\infty}\frac1n\log_2(\nu_{n,1}).$$ Is $1<\lambda_1<\frac32$?

$\endgroup$
9
  • 2
    $\begingroup$ Erm... If you just choose $F$ randomly, then the typical size of the sum over $F$ is $2^{n/2}$ for each $j$, so you get $\nu_{n,p}$ comparable to the trivial Holder bound. Whether the comparability constant is above $1/2$ or not for $p=1$ may be harder to determine, but do you really care? $\endgroup$
    – fedja
    Nov 29, 2021 at 23:50
  • 2
    $\begingroup$ Maximising the function $\nu_{n, 1}$ is equivalent to maximising $\sum_{H} |X| - 2|X \cap H|$ where $X \subseteq \mathbb{F}_{2}^{n}$, and the sum is over hyperplanes of $\mathbb{F}_{2}^{n}$. So the function is large when $X$ is 'unbalanced' with respect to many hyperplanes. Perhaps there is a construction for such objects in the finite geometry literature? $\endgroup$ Nov 30, 2021 at 1:56
  • $\begingroup$ @fedja In fact, for my purposes it would be nice to have affirmative answer to Problem 2. Could you prove that there exists $\varepsilon>0$ such that for a sufficiently large $n$ and randomly chosen $F$ the probablity to have $\sum_{j\in 2^n}|\sum_{i\in F}a_{i,j}|>\varepsilon 2^{3n/2}$ is positive? $\endgroup$ Nov 30, 2021 at 6:30
  • 1
    $\begingroup$ It seems the following. We have that $$2^{2^n}\tilde \nu_{n,1}=\sum_{F\subseteq 2^n}\sum_{j\in 2^n}\big|\sum_{i\in F}a_{i,j}\big|=\sum_{j\in 2^n} S_j$$ where $S_j=\sum_{F\subseteq 2^n}\big|\sum_{i\in F}a_{i,j}\big|$. Let $j\in 2^n$ and $j\ne 0$. Since there is exactly $2^{n-1}$ elements $i\in 2^n$ such that $a_{i,j}=1$, $$S_j=\sum_{k=0}^{2^{n-1}} \sum_{\ell=0}^{2^{n-1}} {2^{n-1}\choose k}{2^{n-1}\choose \ell}|k-\ell|.$$ $\endgroup$ Nov 30, 2021 at 11:22
  • 1
    $\begingroup$ @AlexRavsky Sasha, thank you for your comment. Everything works and computer calculations using your formula agree with known values of $\tilde \nu_{n,1}$ for $n\le 4$. $\endgroup$ Nov 30, 2021 at 13:14

1 Answer 1

2
$\begingroup$

$\newcommand{\tnu}{\tilde\nu}$Continuing Alex Ravsky's comment, we have \begin{equation*} 2^{2^n}\tnu_{n,1}=\sum_{j=0}^{2^n-1}S_j=S_0+(2^n-1)S_1, \tag{1} \end{equation*} where \begin{equation*} S_j:=\sum_{F\subseteq[2^n]}\Big|\sum_{i\in F}a_{i,j}\Big|, \end{equation*} and for each $j\ne0$ \begin{equation*} S_j=S_1=\sum_{k,l=0}^M\binom Mk \binom Ml |k-l|, \end{equation*} where \begin{equation*} M=M_n:=2^{n-1}. \tag{2} \end{equation*}

So, \begin{equation*} S_1=2^{2M} E|K-K'|, \tag{3} \end{equation*} where, for each natural $n$, $K=K_n$ and $K'=K'_n$ are independent random variables (r.v.'s) each with the binomial distribution with parameters $M,1/2$. By the central limit theorem, the distribution of $(K-M/2)/\sqrt{M/4}$ converges weakly to the standard normal distribution (as $n\to\infty$), and hence $V_n:=(K-K')/\sqrt{M/4}$ converges in distribution to $Z-Z'$, where $Z$ and $Z'$ are independent standard normal r.v.'s.

Also, $EV_n^2=2<\infty$ for all $n$ and hence, by the de la Vallée-Poussin theorem, the $V_n$'s are uniformly integrable.

Therefore (see e.g. Theorem 3.5, p. 31), $E|V_n|\to E|Z-Z'|$ and hence, by (3), \begin{equation*} S_1\sim 2^{2M} \sqrt{\frac M4}\;E|Z-Z'|=2^{2M} \sqrt{\frac M4}\;\frac2{\sqrt\pi}. \tag{4} \end{equation*}

Also, \begin{equation*} S_0=\sum_{F\subseteq[2^n]}\sum_{i\in F}1=\sum_{k=0}^{2M}\binom{2M}k k=2^{2M}M. \tag{5} \end{equation*} Collecting the pieces (1), (5), (4), and (2), we finally get \begin{equation*} \tnu_{n,1}\sim\frac1{\sqrt{2\pi}}\,2^{3n/2}. \end{equation*}

$\endgroup$
1
  • $\begingroup$ Thank you very much for the answer! By the way, $\frac1{\sqrt{2\pi}}= 0.398942...$ which agrees with calculations of initial values of $\tilde\nu_{n,1}$. $\endgroup$ Nov 30, 2021 at 14:17

Your Answer

By clicking “Post Your Answer”, you agree to our terms of service and acknowledge you have read our privacy policy.

Not the answer you're looking for? Browse other questions tagged or ask your own question.